m-saghaei

New Member
ارسال ها
338
لایک ها
258
امتیاز
0
پاسخ : ماراتن نظریه ی اعداد (سطح ممتاز)

نه غلطه! يه راهنمايي مي كنم شما بايد ثابت كنيد
اوله نه
.
اگه فرض کنیم
و با توجه به اینکه و
پس نتیجه میشه
که تناقضه.

پس
یعنی
که اگه تو صورت مسئله بزاریم درمیاد
که اینم به ازای p>5 جواب نداره و اگه
رو چک کنیم

همون جوابای قبلی به دست میان!

دیگه اینو مطمئنم!:4:
 
آخرین ویرایش توسط مدیر

AHZolfaghari

Well-Known Member
ارسال ها
935
لایک ها
1,654
امتیاز
93
پاسخ : ماراتن نظریه ی اعداد (سطح ممتاز)

تمام اعداد طبیعی f,g را بیابید که​




:226::226::226::226::226::226:
:226::226::226::226::226::226:
:226::226::226::226::226::226:
:226::226::226::226::226::226:
:226::226::226::226::226::226:
:226::226::226::226::226::226:
:226::226::226::226::226::226:
:226::226::226::226::226::226:
:226::226::226::226::226::226:
:226::226::226::226::226:
​:226:
 
ارسال ها
46
لایک ها
1
امتیاز
8
پاسخ : ماراتن نظریه ی اعداد (سطح ممتاز)

سوال خیلیلی خوبیه تو قسمت اخر یه راهنمایی میکنین؟
 

Dadgarnia

New Member
ارسال ها
1,350
لایک ها
1,127
امتیاز
0
پاسخ : ماراتن نظریه ی اعداد (سطح ممتاز)

سوال خیلیلی خوبیه تو قسمت اخر یه راهنمایی میکنین؟
هنوز يه روز هم از گذاشتن سوال نگذشته اينجا قسمت ممتازه و معلومه كه سوالش توي يه روز حل نميشه پس لطفا بذارين يه هفته حداقل بگذرن بعدش راهنمايي بگيرين!
 

sepidfekr

New Member
ارسال ها
711
لایک ها
637
امتیاز
0

AHZolfaghari

Well-Known Member
ارسال ها
935
لایک ها
1,654
امتیاز
93
پاسخ : ماراتن نظریه ی اعداد (سطح ممتاز)

رفقا ، راجع به این سوالی که گذاشتم نظری ندارید ؟

---- دو نوشته به هم متصل شده است ----

رفقا ، راجع به این سوالی که گذاشتم نظری ندارید ؟
 

aras2213

New Member
ارسال ها
216
لایک ها
228
امتیاز
0
پاسخ : ماراتن نظریه ی اعداد (سطح ممتاز)

رفقا ، راجع به این سوالی که گذاشتم نظری ندارید ؟

---- دو نوشته به هم متصل شده است ----

رفقا ، راجع به این سوالی که گذاشتم نظری ندارید ؟
مجامیع جواب ها:

حلش خیلی طولانی بود،یادمه که باید همه چی رو به پیمانه همه چی میگرفتیم و هی نامساوی میزدیم.
 

AHZolfaghari

Well-Known Member
ارسال ها
935
لایک ها
1,654
امتیاز
93
پاسخ : ماراتن نظریه ی اعداد (سطح ممتاز)

جواب های اقا ارس درست هستش .
راهی که من سراغ دارم اینه که اولش باید ب م م f,g رو بگیریم و ادامه بدیم
 

Dadgarnia

New Member
ارسال ها
1,350
لایک ها
1,127
امتیاز
0
پاسخ : ماراتن نظریه ی اعداد (سطح ممتاز)

یه سوال راحت برای راه افتادن ماراتن:
همه ی اعداد صحیح و مثبت
رابیابید که
.
 

REZA 73

Active Member
ارسال ها
139
لایک ها
184
امتیاز
43
پاسخ : ماراتن نظریه ی اعداد (سطح ممتاز)

یه سوال راحت برای راه افتادن ماراتن:
همه ی اعداد صحیح و مثبت
رابیابید که
.
در حقیقت باید دو معادله زیر رو جواباش رو پیدا کنیم:

برای حالت اول :

پس
. حال اگر
باشد
به پیمانه 5 برابر 4 یا 0 هست. در صورتی که a باید فرد باشد(چون دو مربع کامل با اختلاف 1 وجود ندارند.) پس
به پیمانه 5 برابر 2 یا 3 است.
پس در این حالت معادله جواب غیر بدیهی ندارد. واضح است که بیشمار جواب بدیهی داره.
در حالت دوم هم به طریق مشابه ثابت میشه که
زوجه بعد با بررسی به پیمانه 4 به دست می آید که باز هم جواب غیر بدیهی نداره.


---- دو نوشته به هم متصل شده است ----

سوال بعد:
کوچکترین مقدار
را بیابید که اعداد طبیعی و متمایز
موجود باشند به طوری که :
فکر کنم منبع سوال معلوم باشه تو صورت سوال!!
 

Dadgarnia

New Member
ارسال ها
1,350
لایک ها
1,127
امتیاز
0
پاسخ : ماراتن نظریه ی اعداد (سطح ممتاز)

در حقیقت باید دو معادله زیر رو جواباش رو پیدا کنیم:

برای حالت اول :

پس
. حال اگر
باشد
به پیمانه 5 برابر 4 یا 0 هست. در صورتی که a باید فرد باشد(چون دو مربع کامل با اختلاف 1 وجود ندارند.) پس
به پیمانه 5 برابر 2 یا 3 است.
پس در این حالت معادله جواب غیر بدیهی ندارد. واضح است که بیشمار جواب بدیهی داره.
در حالت دوم هم به طریق مشابه ثابت میشه که
زوجه بعد با بررسی به پیمانه 4 به دست می آید که باز هم جواب غیر بدیهی نداره.


---- دو نوشته به هم متصل شده است ----

سوال بعد:
کوچکترین مقدار
را بیابید که اعداد طبیعی و متمایز
موجود باشند به طوری که :
فکر کنم منبع سوال معلوم باشه تو صورت سوال!!
ميشه جواب هاي بديهي اين سوالو بگين؟ حالت كلي اين سوال هم بيشمار جواب نداره!
بله منبعش معلومه! :4: Information • Art of Problem Solving

---- دو نوشته به هم متصل شده است ----

سوال بعد:
فرض كنيد
عددي اول و
عددي صحيح و مثبت باشد. تعداد چهارتايي هاي
را بيابيد به طوريكه داشته باشيم
و
.
 
آخرین ویرایش توسط مدیر

REZA 73

Active Member
ارسال ها
139
لایک ها
184
امتیاز
43
پاسخ : ماراتن نظریه ی اعداد (سطح ممتاز)

ميشه جواب هاي بديهي اين سوالو بگين؟ حالت كلي اين سوال هم بيشمار جواب نداره!
بله منبعش معلومه! :4: Information • Art of Problem Solving

---- دو نوشته به هم متصل شده است ----

سوال بعد:
فرض كنيد
عددي اول و
عددي صحيح و مثبت باشد. اعداد
را طوري بيابيد كه
و
.


یا


الان از حالت کلی منظورتون دقیقا چی هستش؟؟
 
آخرین ویرایش توسط مدیر

Dadgarnia

New Member
ارسال ها
1,350
لایک ها
1,127
امتیاز
0

aras2213

New Member
ارسال ها
216
لایک ها
228
امتیاز
0
پاسخ : ماراتن نظریه ی اعداد (سطح ممتاز)

سوال بعد:
فرض كنيد
عددي اول و
عددي صحيح و مثبت باشد. تعداد چهارتايي هاي
را بيابيد به طوريكه داشته باشيم
و
.
لم:
معادله
در اعداد صحیح جواب دارد اگر و فقط اگر
.همچنین تعداد جواب های آن برابر در مجموعه
برابر
هه.

حالا میایم رو a_3 حالت بندی میکنیم:

حالت 0 ام-

در این صورت
زمانی برای a_4 جواب دارد که
.پس a_1,a_2 رو به طور دلخواه باید انتخاب کنیم.پس تو این حالت میشه
و a_4 هم به طور یکتا تعیین میشه.
.
.
.
.
حالت i ام-

مثل قبل نتیجه میشه که
.پس باید
و با انتخاب a_1 برای انتخاب a_2
حالت به وجود میاد.همچنین برای انتخاب a_4 ،
حالت داریم.پس در این حالت میشه
.

پس در کل میشه
که این هم میشه
.

:7:

---- دو نوشته به هم متصل شده است ----

سوال بعد:فرض کنید
یه چندجمله ای با ضرایب صحیح باشه.ثابت کنید تابع
از اعداد صحیح به همین مجموعه وجود ندارد که تعداد x هایی که در
صدق میکنند،برابر با
باشد،برای هر n طبیعی.(
این جا یعنی n بار ترکیب تابع
)
 
آخرین ویرایش توسط مدیر

Dadgarnia

New Member
ارسال ها
1,350
لایک ها
1,127
امتیاز
0
پاسخ : ماراتن نظریه ی اعداد (سطح ممتاز)

سوال بعد:فرض کنید
یه چندجمله ای با ضرایب صحیح باشه.ثابت کنید تابع
از اعداد صحیح به همین مجموعه وجود ندارد که تعداد x هایی که در
صدق میکنند،برابر با
باشد،برای هر n طبیعی.(
این جا یعنی n بار ترکیب تابع
)
اين سوال مشكل نداره؟ الان مثلا اگه قرار بديم
به وضوح تابع
شرايط سوالو داره!
 

حمید آنالیز

Well-Known Member
ارسال ها
1,351
لایک ها
1,322
امتیاز
113
پاسخ : ماراتن نظریه ی اعداد (سطح ممتاز)

بله دیگه بیش از یه هفته گزشته لطفا جوایشو بزارین انگاری هیشکی حل نکرده!
 

Dadgarnia

New Member
ارسال ها
1,350
لایک ها
1,127
امتیاز
0
پاسخ : ماراتن نظریه ی اعداد (سطح ممتاز)

بله دیگه بیش از یه هفته گزشته لطفا جوایشو بزارین انگاری هیشکی حل نکرده!
نه سوال خوبیه یه هفته دیگه لطفا صبر کنین اگه حل نشد یه راهنمایی براش بذارین.
 

Dadgarnia

New Member
ارسال ها
1,350
لایک ها
1,127
امتیاز
0
پاسخ : ماراتن نظریه ی اعداد (سطح ممتاز)

كمك كنيد دوباره ماراتن رو راه بندازيم.
لينك سوال قبل: Community - Art of Problem Solving
سوال بعد:
همه ي اعداد طبيعي
را بيابيد به طوريكه دو شرط زير (همزمان) برقرار باشند:
الف)
يه عدد اوله كه
.
ب) عدد اول
و عدد نامنفي
وجود دارند كه
.
 
بالا